IFRAME SYNC
IFRAME SYNC
IFRAME SYNC
IFRAME SYNC

Proving $\;\ln k \geq \int_{k-\frac{1}{2}}^{k+ \frac{1}{2}}\ln x dx$

I'm trying to prove $$\ln k \geq \int_{k-\frac{1}{2}}^{k+ \frac{1}{2}}\ln x dx$$

In other words, I'm trying to show why the area of the rectangle with height $\ln k$ and width $1$ bounds the area under the graph of $f(x)=\ln x$ in the interval $[k-\frac{1}{2},k+\frac{1}{2}].$

I tried to integrate but got stuck. Any ideas for an elegant proof for this?



from Hot Weekly Questions - Mathematics Stack Exchange

Post a Comment

[blogger]

Contact Form

Name

Email *

Message *

copyrighted to mathematicianadda.com. Powered by Blogger.
Javascript DisablePlease Enable Javascript To See All Widget

Blog Archive